Dealing with Imaginary Values in the Expected Momentum Calculation

In summary: I don't even know what to say...In summary, the expected momentum of a system in a particular state is always zero if the wavefunction is purely real.
  • #1
Domnu
178
0
This is more of a general question... let's say we have a wavefunction [tex]\psi[/tex], and we want to find the expected momentum, [tex]\langle p \rangle [/tex], of the state. Isn't this just

[tex]\langle p \rangle = \int_{-\infty}^{\infty} \psi^* i\hbar \frac{\partial \psi}{\partial x}[/tex]​

Now, if the wavefunction happened to be purely real, wouldn't the momentum yield an imaginary value? How do we remedy this? I know that we can expand out [tex]\psi[/tex] in terms of the momentum eigenstates and go from there, but is there any way to just use the above?
 
Physics news on Phys.org
  • #2
Domnu said:
Now, if the wavefunction happened to be purely real, wouldn't the momentum yield an imaginary value?

Yes it would.

Domnu said:
How do we remedy this

I think we set <p>=0 , since p is observable (it must be real)
 
  • Like
Likes K448
  • #3
So whenever [tex]\psi[/tex] is a real function the expected value of the momentum is almost always 0? Would this imply that the probability distribution of the momentum is symmetric about the y axis?
 
  • #4
Can you think of any situations where the wavefunction would be purely real? I don't think there can be, because if Psi is purely real then the LHS of the schrodinger equation is imaginary, but the RHS is real. (Where the SE is in the form here http://en.wikipedia.org/wiki/Schrödinger_equation)
 
  • #5
We could have a wavefunction independent of time, so the left hand side of the time-dependent Schroedinger equation goes to zero.

Edit: Either this, or we can consider the wavefunction at a particular time, say t = 0. Here, the left hand side of the Schroedinger equation goes to zero.
 
  • #6
True. But we know
[tex]<p>=\frac{\partial <x>}{\partial t} = -\int_{-\infty}^{\infty} \psi^* i\hbar \frac{\partial \psi}{\partial x}dx[/tex]

Since [itex]\psi[/tex] is time indepdent, so too must be <x>, so the LHS is zero. That means the RHS must be 0, or [itex]\psi[/tex] must be position independent as well. In other words, [itex]\psi[/itex] must be constant. But constants are not normalizable, so unless I'm missing something, purely real functions cannot solve the SE.

Edit: Unless there are non constants functions f such that f(x)f'(x) = -f(-x)f'(-x), then the RHS could be zero that way... Might have to think about this.
 
Last edited:
  • #7
I'm extremely confused :frown: So you're saying that there is no such thing as a pure real wavefunction? What about the eigenstates of a particle in box? These are purely real...
 
  • #8
That is for the time-independent SE though, right?. The time-dependent part is still complex. And the expected value of the momentum is found using the full solution (time independent * time dependent).
 
  • #9
Okay, so what I meant was that if we have a particular state at t = 0 and the momentum is uncertain, at t = 0, what would the expected momentum be? I don't have any experience with working with the time dependent Schroedinger equation.
 
  • #10
If the time-independent wavefunction is purely real, then [itex]\left<p\right>[/itex] will be zero. To prove this, integrate by parts:

[tex]\int_{-\infty}^{\infty}dx~\psi\frac{d\psi}{dx} = \left.\psi^2\right|^{\infty}_{-\infty} - \int_{-\infty}^{\infty}dx~\frac{d\psi}{dx}\psi[/tex]

The boundary term evaluates to zero, and what you're left with is an expression like A = -A, which of course means A = 0.

Note that the time-dependent part just contributes phase factors which will cancel out (as long as the Hamiltonian is time-independent):

[tex]\left<\hat{p}(t)\right> = \left<\psi\right|e^{i\mathcal{H}t/\hbar}\hat{p}e^{-i\mathcal{H}t/\hbar}\left|\psi\right> = \left<\psi\right|e^{iEt/\hbar}\hat{p}e^{-iEt/\hbar}\left|\psi\right> = \left<\psi\right|\hat{p}\left|\psi\right>[/tex]

However, this isn't the whole story. If your system is in a particular state, then that's true, but in general,

[tex]\left|\psi\right> = \sum_{n}c_ne^{iE_nt/\hbar}\left|\phi_n\right>[/tex]

that is, the system is really in a superposition of states. In this case, what I wrote above doesn't work, because that assumed [itex]\psi = \phi e^{iEt/\hbar}[/itex]. With this superposition of states the time dependence will not cancel out (except at t = 0), and so the wavefunctions are complex.
 
Last edited:
  • #11
Okay, so let me sort of see if I understand this now... let's say we have a wavefunction [tex]\psi = \sqrt{2} \sin (10 \pi x)[/tex]. This is clearly not an eigenfunction of the momentum operator, but is an eigenfunction for the energy operator (for the particle in a box scenario... here, we have a = 1, n = 10). Now, if we find the expected momentum from this, we get zero:

[tex]\langle p \rangle = \int \psi^* \hat{p} \psi dx = 0[/tex]​

Okay, so now let's try to do this another way... let's express [tex]\psi[/tex] in terms of the eigenstates of the momentum operator,

[tex]\phi_n = \frac{1}{\sqrt{2\pi}} e^{ikx}[/tex]​

We have that

[tex]\psi = \frac{1}{\sqrt{2\pi}} \int_{-\infty}^{\infty} b(k) e^{ikx} dx[/tex]​

for complex valued [tex]b(k)[/tex]. We can invert this and get (note that the bounds for the integral change to [0,a] since the wavefunction is zero everywhere else excepting for from x = 0 to a...

[tex]b(k) = \frac{1}{\sqrt{2\pi}} \int_{0}^a \psi e^{-ikx} dx[/tex]​

After we substitute in [tex]\psi[/tex], we find that (after calculations...),

[tex]|b(k)|^2 = \frac{200\pi (1- \cos k)}{(k-10\pi)^2 (k+10\pi)^2}[/tex]​

Now, here's my argument: does this mean that

[tex] \int_{-\infty}^{\infty} k|b(k)|^2 dk = 0[/tex]​

?

Edit: YES! It works! This is simply because

[tex]\frac{200\pi (1- \cos k)}{(k-10\pi)^2 (k+10\pi)^2}[/tex]​

is an even function! Thanks very much guys... I really appreciate it. My understanding of QM just went up a bunch! :smile: :smile: :smile: Just so you know... if you wanted to know why the evenness of this function caused the integral to be 0, if you multiply x by f(x) where f(x) is even, x*f(x) becomes odd. If you take the integral from -t to t of an odd function, you always get 0.
 
Last edited:
  • #12
Domnu said:
Now, here's my argument: does this mean that

[tex] \int_{-\infty}^{\infty} k|b(k)|^2 dk = 0[/tex]​

?

Edit: YES! It works! This is simply because

[tex]\frac{200\pi (1- \cos k)}{(k-10\pi)^2 (k+10\pi)^2}[/tex]​

is an even function! Thanks very much guys... I really appreciate it. My understanding of QM just went up a bunch! :smile: :smile: :smile: Just so you know... if you wanted to know why the evenness of this function caused the integral to be 0, if you multiply x by f(x) where f(x) is even, x*f(x) becomes odd. If you take the integral from -t to t of an odd function, you always get 0.

Careful there - that's only strictly true if the integrand tends to zero at [itex]\pm \infty[/itex], otherwise the integral is an indeterminate form. As k gets large your integrand goes as (1-cosk)/k^3, so it does indeed vanish at the boundary and you get zero for the integral.
 
  • #13
Mute said:
Careful there - that's only strictly true if the integrand tends to zero at [itex]\pm \infty[/itex], otherwise the integral is an indeterminate form. As k gets large your integrand goes as (1-cosk)/k^3, so it does indeed vanish at the boundary and you get zero for the integral.

Well, yes, heheh I forgot to mention that :biggrin: But yea... it goes to zero since the denominator grows arbitarily large while the numerator stays fixed within 0 and 400 pi.
 

1. What is the definition of "Expected Value of Momentum"?

The expected value of momentum is a measure of the average momentum of a system or object, taking into account all possible outcomes and their respective probabilities.

2. How is the expected value of momentum calculated?

The expected value of momentum is calculated by multiplying the momentum of each possible outcome by its respective probability, and then summing these values together.

3. What is the significance of the expected value of momentum?

The expected value of momentum can be used to predict the behavior of a system or object in terms of momentum. It can also provide insight into the overall direction and magnitude of momentum in a given situation.

4. How does the expected value of momentum relate to the concept of uncertainty?

The expected value of momentum takes into account all possible outcomes and their respective probabilities, thus incorporating uncertainty into its calculation. It provides a measure of the average momentum that can be expected from a system or object, despite the presence of uncertainty in its behavior.

5. Can the expected value of momentum be negative?

Yes, the expected value of momentum can be negative. This would indicate that, on average, the momentum of the system or object is in the opposite direction of what is expected or desired.

Similar threads

  • Advanced Physics Homework Help
Replies
10
Views
554
Replies
3
Views
1K
  • Advanced Physics Homework Help
Replies
30
Views
1K
  • Advanced Physics Homework Help
Replies
8
Views
3K
Replies
14
Views
1K
  • Advanced Physics Homework Help
Replies
2
Views
2K
  • Advanced Physics Homework Help
Replies
1
Views
1K
  • Advanced Physics Homework Help
Replies
1
Views
2K
  • Advanced Physics Homework Help
Replies
3
Views
1K
Back
Top